Quantcast
  • Register
PhysicsOverflow is a next-generation academic platform for physicists and astronomers, including a community peer review system and a postgraduate-level discussion forum analogous to MathOverflow.

Welcome to PhysicsOverflow! PhysicsOverflow is an open platform for community peer review and graduate-level Physics discussion.

Please help promote PhysicsOverflow ads elsewhere if you like it.

News

PO is now at the Physics Department of Bielefeld University!

New printer friendly PO pages!

Migration to Bielefeld University was successful!

Please vote for this year's PhysicsOverflow ads!

Please do help out in categorising submissions. Submit a paper to PhysicsOverflow!

... see more

Tools for paper authors

Submit paper
Claim Paper Authorship

Tools for SE users

Search User
Reclaim SE Account
Request Account Merger
Nativise imported posts
Claim post (deleted users)
Import SE post

Users whose questions have been imported from Physics Stack Exchange, Theoretical Physics Stack Exchange, or any other Stack Exchange site are kindly requested to reclaim their account and not to register as a new user.

Public \(\beta\) tools

Report a bug with a feature
Request a new functionality
404 page design
Send feedback

Attributions

(propose a free ad)

Site Statistics

205 submissions , 163 unreviewed
5,047 questions , 2,200 unanswered
5,345 answers , 22,709 comments
1,470 users with positive rep
816 active unimported users
More ...

  A simple model that exhibits emergent symmetry?

+ 4 like - 0 dislike
1734 views

In a previous question Emergent symmetries I asked, Prof.Luboš Motl said that emergent symmetries are never exact. But I wonder whether the following example is an counterexample that has exact emergent spin rotational symmetry.

Just consider the simplest Ising model for two spin-1/2 system $H=\sigma_1^z\sigma_2^z$, it has two ground states, one of them is spin-singlet $|\uparrow\downarrow> -|\downarrow\uparrow> $ which possesses spin rotational symmetry, while the original Hamiltonian explicitly breaks it.

And I want to know if anyone knows some simple examples that all of the ground states have the emergent symmetry while the Hamiltonian doesn't have?

By the way,I remember that Prof.Xiao-gang Wen has said, a key difference between "topological degeneracy" and "ordinary degeneracy" is that the topological degeneracy is generally approximate while the ordinary degeneracy is exact. If the emergent symmetries are generally approximate, whether are there some connections between the topological degeneracy and emergent symmetries?

Thanks in advance.

This post imported from StackExchange Physics at 2014-03-09 08:46 (UCT), posted by SE-user K-boy
asked Mar 22, 2013 in Theoretical Physics by Kai Li (980 points) [ no revision ]
@user10001:Thanks for your corrections.

This post imported from StackExchange Physics at 2014-03-09 08:46 (UCT), posted by SE-user K-boy
How about this one: $H=S_1^z S_2^z+\frac{1}{4}(S_1^z+S_2^z)$. The second term breaks the degeneracy of the excited states in your model. The operator $S_1^+S_2^++S_1^-S_2^-$ does not commute with the Hamiltonian, but the two ground states are its eigenstates. My logic is to construct a symmetry operator that commutes with the Hamiltonian only in the subspace of the ground states.

This post imported from StackExchange Physics at 2014-03-09 08:46 (UCT), posted by SE-user Tengen
I'm thinking about the one dimensional antiferromagnetic Haldane chain (with open boundary condition). The gound state is 4-fold degenerate only in thermodynamic limit. That means the energy difference between a state with total spin 0 and a state with total spin 1 becomes zero in thermodynamic limit. I'm wondering how to get it from symmetry point of view.

This post imported from StackExchange Physics at 2014-03-09 08:46 (UCT), posted by SE-user Tengen
@Tengen:The ground states in your model are the same as mine, I wonder whether there exist simple models whose ground states all have emergent symmetries.

This post imported from StackExchange Physics at 2014-03-09 08:46 (UCT), posted by SE-user K-boy
My model is an example. The gound state subspace is spanned by $|\uparrow\downarrow\rangle$ and $|\downarrow\uparrow\rangle$. They all have the symmetry as I defined above. But only a one dimentional subspace spanned by $|\uparrow\downarrow\rangle-|\downarrow\uparrow\rangle$ has spin rotation symmetry.

This post imported from StackExchange Physics at 2014-03-09 08:46 (UCT), posted by SE-user Tengen
@Tengen:Oh,I got what you mean now. You're right,the two ground states are definitely eigenstates of the symmetry operator you defined,but with eigenvalues zeros, which means that the ground states would disappear once the symmetry operator acts on them.So in this case can we still say that the ground states are symmetric under the operation you defined?

This post imported from StackExchange Physics at 2014-03-09 08:46 (UCT), posted by SE-user K-boy
@Tengen:What more important, I think the "symmetry" operator you defined in fact doesn't represent any symmetry, because it can not even be inversed, which means that it can not be an unitary or antiunitary operator. And its physical meaning is unclear.

This post imported from StackExchange Physics at 2014-03-09 08:46 (UCT), posted by SE-user K-boy
@Tengen:Maybe I misunderstood your explanation.Let A be the operator you defined, then exp(i A ) can be a symmetry operator, did you mean this? If so ,then what's the physical meaning of it?

This post imported from StackExchange Physics at 2014-03-09 08:46 (UCT), posted by SE-user K-boy
We say a subspace has a symmetry if it support a representation of the symmetry group. All we are talking about here is the generator of the symmetry transformation, not the group itself. Zero eigenvalue is not a problem (you can plus any nonzero number to the symmetry operator if you like). Think about the spin rotational symmetry: the total spin can also be zero.

This post imported from StackExchange Physics at 2014-03-09 08:46 (UCT), posted by SE-user Tengen
Yes, you are right. No physical meanings maybe. I just constructed it to fulfil your requirement.

This post imported from StackExchange Physics at 2014-03-09 08:46 (UCT), posted by SE-user Tengen
@Tengen:Yeah, I agree with you now, I made a misunderstanding. So do you know the physical meaning of it?

This post imported from StackExchange Physics at 2014-03-09 08:46 (UCT), posted by SE-user K-boy
@Tengen: Ok, I see, you really give us an example of what I want, although its physical meaning is temporarily unclear. Thank you all the same.

This post imported from StackExchange Physics at 2014-03-09 08:46 (UCT), posted by SE-user K-boy
You can construct them easily as many as you like. As I said before, all you need is to construct a symmetry operator that commutes with the Hamiltonian only in the subspace of the ground states.

This post imported from StackExchange Physics at 2014-03-09 08:46 (UCT), posted by SE-user Tengen
@Tengen: Yeah, that's right

This post imported from StackExchange Physics at 2014-03-09 08:46 (UCT), posted by SE-user K-boy
@Tengen: For the Haldane chain you mentioned, I didn't know much about it. But I want to know if you take periodic boundary conditions, are the ground states exactly degenerate for finite lattice sites? What causes the ground state degeneracy in Haldane chain, topology or symmetry of the system? If it's the topology, whether these topological ground states are protected by some symmetries?

This post imported from StackExchange Physics at 2014-03-09 08:46 (UCT), posted by SE-user K-boy
The ground state is nondegenerate if you take periodic boundary condition. It is an example of symmetry protected topological state. The symmetry is the spin rotational symmetry SO(3) for integral spin.

This post imported from StackExchange Physics at 2014-03-09 08:46 (UCT), posted by SE-user Tengen

1 Answer

+ 3 like - 0 dislike

The simplest model is the spin-1/2 chain with Majumdar–Ghosh interaction: $$H=\sum_i P_{3/2}(i-1,i,i+1),$$ where $P_{3/2}(i,j,k)$ is the projection operator that projects a state onto the subspace with total spin-3/2 on sites $i,j,k$. The ground states are two dimer states (see the figure on wikipedia Majumdar–Ghosh model): $$|\psi_1\rangle=\prod_i|\mathrm{singlet}\rangle_{2i,2i+1},$$ $$|\psi_2\rangle=\prod_i|\mathrm{singlet}\rangle_{2i-1,2i}.$$

If we define the symmetry transformation $U(i,j)=\exp(ia_{ij}P_0(i,j))$ where $P_0(i,j)$ is the singlet projection operator, then $$U(2i,2i+1)|\psi_1\rangle=\exp(i a_{2i,2i+1})|\psi_1\rangle,$$ $$U(2i-1,2i)|\psi_2\rangle=\exp(i a_{2i-1,2i})|\psi_2\rangle,$$ for any $i$. In other words, $|\psi_1\rangle$ supports a one dimensional representation of the group $U(2i,2i+1)$ (any $i$) which is not a symmetry of the original Hamiltonian. Similar for $|\psi_2\rangle$. It is exactly those emergent symmetries that make this model soluble.

More sophisticated examples can be found here: 0207106.

This post imported from StackExchange Physics at 2014-03-09 08:46 (UCT), posted by SE-user Tengen
answered Apr 1, 2013 by Tengen (105 points) [ no revision ]
Thanks for your answer.But I can't understand your explanation clearly cause your language seems a little hard to me. Do you mean that the two dimer ground states of MG model both have the same symmetry $ U(i,j) $ while the Hamiltonian $H$ doesn't have ?

This post imported from StackExchange Physics at 2014-03-09 08:46 (UCT), posted by SE-user K-boy
I've modified the answer, please have a look.

This post imported from StackExchange Physics at 2014-03-09 08:46 (UCT), posted by SE-user Tengen
Ok. How about the superposition of the two dimer states?For example, whether the ground state $\psi=\lambda_1\psi_1+\lambda_2\psi_2$ is still a eigenstate of $U(i,j)$ ? Thanks a lot.

This post imported from StackExchange Physics at 2014-03-09 08:46 (UCT), posted by SE-user K-boy
And I like your last sentence "It is exactly those emergent symmetries that make this model soluble" very much, since this maybe one of the facts that would make emergent symmetries important in physics.

This post imported from StackExchange Physics at 2014-03-09 08:46 (UCT), posted by SE-user K-boy
Nope. There are $N$ $U(1)$ groups: $U(i,i+1), i=1,2,...,N$. $|\psi_1\rangle$ is invariant under the actions of half of there groups, and $|\psi_2\rangle$ the other half.

This post imported from StackExchange Physics at 2014-03-09 08:46 (UCT), posted by SE-user Tengen

Your answer

Please use answers only to (at least partly) answer questions. To comment, discuss, or ask for clarification, leave a comment instead.
To mask links under text, please type your text, highlight it, and click the "link" button. You can then enter your link URL.
Please consult the FAQ for as to how to format your post.
This is the answer box; if you want to write a comment instead, please use the 'add comment' button.
Live preview (may slow down editor)   Preview
Your name to display (optional):
Privacy: Your email address will only be used for sending these notifications.
Anti-spam verification:
If you are a human please identify the position of the character covered by the symbol $\varnothing$ in the following word:
p$\hbar\varnothing$sicsOverflow
Then drag the red bullet below over the corresponding character of our banner. When you drop it there, the bullet changes to green (on slow internet connections after a few seconds).
Please complete the anti-spam verification




user contributions licensed under cc by-sa 3.0 with attribution required

Your rights
...